You are on page 1of 12

www.gradeup.

co

1. Consider a system of linear equations:


x  2 y  3z  1 4. Let z  x  iy be a complex variable. Consider that
x  3 y  4 z  1 and contour integration is performed along the unit circle in
anticlockwise direction. Which one of the following
2 x  4 y  6 z  k
statements is NOT TRUE?
The value of k for which the system has infinitely many z
solution is ____________. A. The residue of at z = 1 is 1/2
A. 4 B. 2 z2 1
C. 8 D. 5
Answer ||| B B.
Solution |||
x  2 y  3z  1
x  3 y  4 z  1 and C.
2 x  4 y  6 z  k D. z (complex conjugate of z) is an analytical function
Answer ||| D
 1 2 3 :  1 Solution ||| f ( z )  z  x  iy
[ A : B]   1 3 4 : 1  u  x v  y
 2 4 6 : K   ux  1 vx  0
R2  R2  R1 , R3  R3  2R1 uy  0 v y  1
1 2 3 :  1  u x  v y i.e. C - R not satisfied
0 1 1 : 2 
  ⇒ z is not analytic function.
0 0 0 : K  1
For infinite may solution 1 
ρ(A : B) = ρ(A)  
5. The value of p such that the vector 2 is an
= r < number of variables  
 3 
ρ(A : B) = 2
k–2=0 4 1 2
k=2 
eigenvector of the matrix p 2 1  is ________.

2. A function f ( x )  1  x 2  x 3 is defined innte closed 14 4 10 
interval [–1, 1]. The value of x, in the open interval (–1, A. 17 B. 28
1) for which the mean value theorem is satisfied, is C.21 D.18
A. –1/2 B. –1/3 Answer ||| A
C. 1/3 D. 1/2 Solution ||| AX   X
Answer ||| B  4 1 2  1  1 
Solution ||| Since f(1) ≠ f{-1), Roll's mean value  p 2 1   2    2
theorem does not apply.      
By Lagrange mean value theorem 14 4 10  3  3
f (1)  f (1) 2  12  1 
f '( x)   1
1  (1) 2  p  7     2
   
2x  3x2  1  36   3 
1
x  1,  p7
3  2  p  17
12
x lies in (-1, 1)
1 6. In the circuit shown, at resonance, the amplitude of
x
3 the sinusoidal voltage (in Volts) across the capacitor is
3. Suppose A and B are two independent events with ________.
probabilities P(A) ≠ 0 and P(B) ≠ 0. Let A and B be
their complements. Which one of the following
statements is FALSE?
A. P( A  B)  P( A) P( B) B. P( A / B)  P( A)
A. 45 B. 12
C. P( A  B)  P( A)  P(B) D. P( A  B)  P( A)  P(B) C.24 D.25
Answer ||| C Answer ||| D
Solution ||| P(A ∪ B) = P(A) + F(B) - P(A ∩ B) 10
Since P(A ∩ B) = P(A) p(B) Solution ||| At resonance I   2.5 A
4
(not necessarily equal to zero).
So, P(A ∪ B) = P(A) + P(B) is false.

1|Page
www.gradeup.co

1
  105 rad / sec
3 6
0.1 10  10
1 1
XC    10
C 105 1106
X C  IX C  10  2.5  25Volts
Current flowing through both the parallel 4  will be I.
7. In the network shown in the figure, all resistor s are So, V2  4( I  I  2 I )  4 I by KVL
identical with R = 300 Ω. The resistance Rab (in Ω) of the
network is ___________.
I  I  2I  5 by KVL
5
I A
4
45
V2  4  5   25V
4
45
V1  4 I   5V
4
A. 125 B. 231
C.100 D.200 9. A region of negative differential resistance is observed
Answer ||| C in the current voltage characteristics of a silicon PN
Solution ||| junction if
Modifying the given circuit A. both the P-region and the N-region are heavily doped
B. the N-region is heavily doped compared to the P-
region
C.the P-region is heavily doped compared to the N-region
D.an intrinsic silicon region is inserted between the P-
region and the N-region
Answer ||| A
Solution ||| A region of negative differential resistance is
observed in the current voltage characteristics of a silicon
PN junction if both the P-region and the N-region are
heavily doped.

10. A silicon sample is uniformly doped with donor type


impurities with a concentration of 1016 /cm3. The
electron and hole mobilities in the sample are 1200
cm2/V-s and 400 cm2 /V-s respectively.
Assume complete ionization of impurities. The charge of
an electron is 1.6 × 10–19 C. The resistivity of the sample
(in Ω-cm) is _________.
A. 0.345 B. 0.520
C.0.234 D.0.423
Answer ||| B
1 Solution |||
 1 1 1 1  R 300
Rab          100 1 1
 2 R R R 2 R 3 3 P 
N N D q n
8. In the given circuit, the values of V1 and V2 1
  0.52  cm
respectively are: 1016  1.6  1019  1200

11. For the circuit with ideal diodes shown in the figure,
the shape of the output (υout) for the given sine wave
input (υin) will be

A. 5 V, 25 V B. 10 V, 30 V
C.15 V. 35 V D.0 V, 20 V
Answer ||| A
Solution |||

A.
2|Page
www.gradeup.co

A. 0.3 B. 0.45
C.0.9 D.3
Answer ||| C
Solution |||
B. vc (0 )  0 V
vc (0 )  0 V
vc ()  0 V
C.
Time constant
  RC  120  0.1106
vc (t )  3  (0  3) e  t /  3  (1  e  t / )
D.
Cdvc (t ) 0.1106  3  et /
I c (t )  
Answer ||| C dt 
6  t /
Solution ||| The circuit can be redrawn as 0.110  3e 1
  et /
120  0.1106 40

Energy  VIdt 
0

1 3  3
  3.  e  t /     e  t /   12  06  0.9 J
0
40 40 0
40
During positive pulse, both diodes are forward biased.
During negative pulse, both diodes are reverse biased. 14. In an 8085 microprocessor, the shift registers which
So, Vo = 0V store the result of an addition and the overflow bit are,
respectively
12. In the circuit shown below, the Zener diode is ideal A. B and F B. A and F
and the Zener voltage is 6V. The output voltage V0 (in C.H and F D.A and C
volts) is ____________. Answer ||| B
Solution ||| Shift register are accumulator and flag
register (A & F).
Hence B. is the correct option.

15. A 16 Kb (=16,384 bit) memory array is designed as a


square with an aspect ratio of one (number of rows is
equal to the number of columns). The minimum number
A. 6 B. 8 of address lines needed for the row decoder is
C.4 D.2 __________ .
Answer ||| B A. 5 B. 7
Solution ||| C.8 D.9
Answer ||| B
Solution ||| Memory size = 16 kB = 214 bits
Number of address lines = Number of data lines
From
2n • 2n = 214
From n = 7

16. Consider a four bit D to A converter. The analog


Vx Vx  Vy value corresponding to digital signals of values 0000 and
  0.5Vx  5 A ...(i)
20 10 0001 are 0 V and 0.0625 V respectively. The analog
V y  0.25Vx . . . (ii) value (in Volts) corresponding to the digital signal 1111 is
________.
Vx  8Volts A. 0.8724 B. 0.8283
C.0.9375 D.0.9127
13. In the circuit shown, the switch SW is thrown from Answer ||| C
position A to position B at time t = 0. The energy (in µJ) Solution ||| Step size = 0.0625 V
taken from the 3V source to charge the 0.1 µF capacitor Decimal equivalent = 15
from 0 V to 3 V is Analog output = 15 x 0.0625 = 0.9375 Volts

17. The result of the convolution x(–t) * δ (–t – t0) is


A. x(t  t0 ) B. x(t  t0 )
C. x(t  t0 ) D. x(t  t0 )

3|Page
www.gradeup.co

Answer ||| D
Solution ||| x(t )*  (t  t0 )  x(t )*  (t  t0 ) 10( s  1)
21. The polar plot of the transfer function G ( s ) 
 x(t  t0 ) s  10
for 0     will be in the
18. The waveform of a periodic signal x(t) is shown in the A. first quadrant B. second quadrant
figure. C.third quadrant D.fourth quadrant
Answer ||| A
10( s  1)
Solution ||| G ( s ) 
( s  10)
For
  0  M1  1 and 1  0
 t 1    0  M 2  10
A signal g(t) is defined by g (t )  x   . The average
 2  And 2   tan 1   tan 1   0
power of g(t) is __________.
A. 7 B. 2  Polar plot
C.6 D.8
Answer ||| B
 t 1  3
Solution ||| x     (t  1) 1  t  3
 2  2
And T = 8
3 2
1  3 
8 1  2
Average power   (t  1)  dt  2

19. Negative feedback in a closed-loop control system


DOES NOT
A. reduce the overall gain
B. reduce bandwidth
As the zero is nearer to the imaginary axis hence the
C.improve disturbance rejection
direction of polar plot is clockwise.
D.reduce sensitivity to parameter variation
Answer ||| B
22. A sinusoidal signal of 2 kHz frequency is applied to a
Solution ||| Negative Feedback reduces gain but
delta modulator. The sampling rate and step-size Δ of the
Bandwidth is (). So, Negative feedback in a closed-loop
delta modulator are 20, 000 sample per second and 0.1
control system DOES NOT reduce bandwidth.
V, respectively. To prevent slope overload, the maximum
amplitude of the sinusoidal signal (in Volts) is
20. A unity negative feedback system has the open-loop
1 1
K A. B.
transfer function G ( s )  . The value of the 2 
s( s  1)( s  3)
2
gain K (>0) at which the root locus crosses the imaginary C. D.π
axis is ____________. 
A. 500 B. 400 Answer ||| A
C.200 D.300 Solution ||| Slope signal = Slope of delta modulator
Answer ||| B Am (2 f m )  f s
Solution ||| Am (2 2  103 )  20, 000  0.1
G(s) K
T ( s)   1
1  G( s) H ( s) s 2  10s  K Am 
2
Comparing with standard second order
transfer function
23. Consider the signal
n2 s(t )  m(t ) cos(2 f c t )  m
ˆ (t )sin(2 f c t )
T ( s) 
s  2 zn s  n2
2
ˆ
Where m(t ) denotes the Hilbert transform of m(t) and
We have
the bandwidth of m(t) is very small compared to f c . The
 n2  K
signal s(t) is a
And 2n  10 A. high-pass signal
  0.25 (given) B. low-pass signal
C.band-pass signal
10
 n   20 D.double sideband suppressed carrier signal
2  0.25 Answer ||| C
And K  n2  (20) 2  400 Solution ||| Given s(f) is an SSB modulated signal.
Alternate Solution:
4|Page
www.gradeup.co

It is the canonical representation of a bandpass signal. A. (2  t ) et B. (1  2t ) e  t

24. Consider a straight, infinitely long, current carrying C. (2  t ) e  t D. (1  2t ) et


conductor lying on the z-axis. Which one of the following Answer ||| B
plots (111 linear scale) qualitatively represents the Solution |||
dependence of H  oil r, where H  is the magnitude of r2 + 2r + 1 = 0
roots are equal r1 = r2 = - 1
the azimuthal component of magnetic field outside the
So, y = c1e-t + c2t e-t
conductor and r is the radial distance from the
y(0) = C1 = 1
conductor?
y = - C1e-t – C2t e-t + C2e-t
y(0) = - C1 + C2 = 1
C2 = 2
So solution is y = e-t + 2te-t
y = (1 + 2t) e-t.

A. B. 27. A vector is given by .


Which one of the following statements is TRUE?
A. is solenoidal, but not irrotational
B. is irrotational, but not solenoidal
C. is neither solenoidal nor irrotational
C. D. D. is both solenoidal and irrotational
Answer ||| C Answer ||| A
I Solution |||
Solution ||| H   aˆ
2 r
1
 H 
r
Option on C.is satisfied.

25. The electric field component of a plane wave traveling


in a lossless dielectric medium is given by

28. Which one of the following graphs describes the


V/m. The wavelength (in m) foe
function f ( x)  e  (x) ( x 2  x  1) ?
the wave is ___________.
A. 8.89 B. 7.28
C.4.87 D.6.23
Answer ||| A
Solution |||

A.

General form of

Comparing both, we get B.


1

2
2
Since,  

C.
2 2
    2 2 m
 1
2
   8.88m

26. The solution of the differential equation


D.
d2y dy Answer ||| B
2
 2  y  0 with y(0 = y’(0) = 1 is
dt dt Solution ||| f ( x )  e  x ( x 2  x  1)
5|Page
www.gradeup.co

f '( x)  e  x (2 x  1)  e x ( x  x 2 )
 e  x ( x  x 2 )  e x ( x)(1  x)
Putting f’(x) = 0, we get
x = 0, or x = 1
f ''( x)  e  x (1  2 x)  e  x ( x  x 2 )
A. 2.528 B. 4.234
 e  x (1  3 x  x 2 ) C.3.343 D.4.343
At x = 0, f"(x) = 1 (so we have a minimum) Answer ||| A
1 Solution ||| vc (0 )  0 V
At x = 1, f ''( x)  
e vc (0 )  0 V
(so we have a maximum), curve B. shows a single local
At t = ∞
minimum = 0 and a single local maximum at x = 1.

29. The maximum area (in square units) of a rectangle


whose vertices lie on the ellipse x 2  4 y 2  1 is _______.
A. 1 B. 5
C.6 D.8
Answer ||| A 2
vc ()   10
Solution ||| 23
= 4V [By voltage divider]
vc (t )  4[ I  e  t / ]
3 2 5
  Req C  
3 2 6
= 1 sec
vc (1)  4[1  e 1/1 ]  2.528 volts

x2  4 y 2  1 32. In the given circuit, the maximum power (in Watts)


Area of rectangle that can be transferred to the load RL is ________.
 2x.2 y  4xy
Let f = (Area)2=16x2y2

f '( x)  0
1
We get x  A. 3.283 B. 1.649
2
C.2.832 D.3.123
1 Answer ||| B
y
8 Solution |||
1 1
Area  4 xy  4   1
2 8

30. The damping ratio of a series RLC circuit can be


expressed as For maximum power transfer
R 2C 2L RL | RTh || 2 || j2 |
A. B. 2
2L RC 2  j2
  1.414
R C 2 L 2  j2
C. D.
2 L R C 40
Answer ||| C VTh   2  j 2  2.82845
2  j2
R C
Solution ||| Damping ratio 
2 L

31. In the circuit shown, switch SW is closed at t = 0.


Assuming zero initial conditions, the value of vc (t ) (in
Volts) at t = 1 sec is ____________.

2.82845
I  1.0822.5
1.41445  1.414

6|Page
www.gradeup.co

Power = I 2 R  (1.08)2  2  1.649W L p   p D p  100  10 6  36


 0.06cm
33. The built-in potential of an abrupt p-n junction is 0.75
Using the above values, we get hole current density
V. If its junction capacitance (Q) at a reverse bias (VR) of
injected from P region to N region is = 286.17 nA/cm2
1.25 V is 5 pF, the value of CJ (in pF) when VR = 7.25 V
is________.
36. The Boolean expression
A. 3.4 B. 2.5
C.4.7 D.3.5 F ( X , Y , Z )  XY Z  XY Z  XY Z  XYZ converted into the
Answer ||| B canonical product of sum (POS) form is
Solution ||| A. ( X  Y  Z )( X  Y  Z )( X  Y  Z )( X  Y  Z )
1
Cj  B. ( X  Y  Z )( X  Y  Z )( X  Y  Z )( X  Y  Z )
Vbi  VR
C. ( X  Y  Z )( X  Y  Z )( X  Y  Z )( X  Y  Z )
C2 j Vbi  VR1
 D. ( X  Y  Z )( X  Y  Z )( X  Y  Z )( X  Y  Z )
C1 j Bbi  VR2
Answer ||| A
2 C1 j Solution ||| F ( X , Y , Z )  XY Z  XY Z  XY Z  XYZ
C2 j  C1 j   2.5 pF
8 2
So, answer is 2.5.

34. A MOSFET in saturation has a drain current of 1 mA


for VDS = 0.5 V. If the channel length modulation
coefficient is 0.05 V–1, the output resistance (in kΩ) of
the MOSFET is _______.
A. 25 B. 18 F   X  Y  Z   X  Y  Z  X  Y  Z  X  Y  Z 
C.20 D.30 37. All the logic gates shown in the figure have a
Answer ||| C propagation delay of 20 ns. Let A = C = 0 and B = 1 until
Solution ||| Under channel length modulation time t = 0. At t = 0, all the inputs flip (i.e., A = C = 1
I D  I Dsat (1  VDS ) and B = 0) and remain in that state. For t > 0, output Z
= 1 for a duration (in ns) of _________.
dI D 1
   I Dsat
dVDS r0
1 1
r0    20k  A. 20 B. 40
 I Dsat 0.05 103
C.35 D.25
Answer ||| B
35. For a silicon diode with long P and N regions, the Solution |||
accepter and donor impurity concentrations are 1 x 101
cm–3 and 1 × 1015 cm–3, respectively. The lifetimes of
electrons in P region and holes in N region are both 100
µs. The electron and hole diffusion coefficients are 49
cm2/s and 36 cm2/s, respectively. Assume kT/q = 26 mV,
the intrinsic earner concentration is 1× 1010 cm–3, and q
= 1.6 × 10–19 C. When a forward voltage of 208 mV is
applied across the diode, the hole current density (in
nA/cm2) injected from P region to N region is ______.
A. 356.17 B. 156.23
C.286.17 D.432.12
Answer ||| C
Solution ||| The hole current density injected from P
region to N region is given by
qni2 D p   VFB  
exp    1
N D Lp   VT  
Where, Zis ‘1’ for 40 n-sec.
Q = charge on electron
ni = Intrinsic carrier concentration in silicon 38. A 3-input majority gate is defined by the logic
ND = Donor doping function M(a, b,c) = ab + be + ca . Which one of the
DP = Hole diffusion coefficient following gates is represented by the function
Lp = Mean diffusion length of hole
VFB = Forward voltage applied across diode
M (M (a, b, c), M (a, b, c), c) ?
VT = kT/q = 26 mV A. 3-input NAND gate B. 3-input XOR gate
C.3-input NOR gate D.3-input XNOR gate

7|Page
www.gradeup.co

Answer ||| B
Solution ||| M (a, b, c)  ab  bc  ca
M (a, b, c)  ab  bc  ca
 ab.bc.ca
 (a  b)(b  c)(c  a )
M (a, b, c)  ab  bc  ca A. 350 B. 250
C.450 D.125
M (M(a, b, c), M(a, b, c) c)  Answer ||| B
(ab.bc.ca)(ab  bc  ca)  1
  Solution ||| I 50   A  I100 
50
 (ab b c  ca)(c)  (ab.bc.ca)c 
1
(a  b)(b  c)(c  a)(ab bc  ca )  V0  [250(1   )  250(1   )
  100
  abc (a  b)(b  c)  (c  a) c  1
  250  0.25V  250mV
 (a  b)(b  c)(c  a)[ab  bc  ca  c]  abc 100
 (ab  bc  ca)[a  b  c]  abc 41. The circuit shown in the figure has an ideal opamp.
F  abc  bca  cab  abc The oscillation frequency and the condition to sustain the
oscillations, respectively, are
F  A B C
39. For the NMOSFET in the circuit shown, the threshold
voltage is Vth, where Vth > 0. The source voltage VSS is
varied from 0 to VDD. Neglecting the channel length
modulation, the drain current ID as a function of VSS is
represented by

1
A. and R1  R2
CR
1
B. and R1  4 R2
CR
1
C. and R1  R2
2CR
1
D. and R1  4 R2
2CR
Answer ||| D
A. B. 1
Solution ||| ω0 = & (R1 = 5R2)
2RC

C. D.
Answer ||| A
Solution ||| VGS  VDS
Hence MOS transistor is in saturation.
In saturation, 1
I D  k (VGS  V ) 2r  k (VDD  VSS  VT ) 2 Frequency of Wein bridge is ω0 = but here Time
RC
As Vss increases ID decreases (Not linearly because constant is doubled so frequency becomes halved ⇒ ω0 =
square factor) Hence option A. is correct.
1
40. In the circuit shown, assume that the opamp is ideal. 2RC
The bridge output voltage V0 (in mV) for δ = 0.05 is 1
__________. z1 = 2R +  2  R  jR 
jC

8|Page
www.gradeup.co

R
1 R 1
  2R  abc 
2 jC 2 j  3 
z2    p  1 W3
2
W31   
1 R  jR 1 2
R 
 q   1 W 1W 2 2 1   a  bW3  cW3 
2 jC    3 3 W3 W3   
3
z2  r  1 W32W32 W31W31   
1 2 1
 a  bW3  cW3 
β= 
z1  z2 5  
 3 
R1 2
Now, A = 5 1 +  5 (R1 = 4R2) Where W3  e
1
j
3
R2
4
j
W32  e 3
42. In the circuit shown, I1 = 80 mA and I2 = 4 mA.
2
Transistors T1 and T2 are identical. Assume that the j

thermal voltage VT is 26 mV at 27°C. At 50°C, the value


W31  e 3

4
of the voltage V12 = V1 – V2 (in mV) is _______. j
W32  e 3

2
j
W34  W31  e 3

c 
  a 
 b 

44. For the discrete-time system shown in the figure, the


poles of the system transfer function are located at
A. 87.14 B. 83.15
C.84.12 D.81.13
Answer ||| B
VBE2
VT
Solution ||| I 2  I s e
VBE2  V2
VBE1
1
A. 2, 3 B. ,3
I1  I s e VT 2
VBE1  V1 1 1 1
C. , D. 2,
v v 2 3 3
I1 1 2

 e VT Answer ||| C


I2 Solution ||| The difference equation of the system
Since VT at 27° is 26 mV then VT at 50° is 27.99 mV. 1 5
Thus x ( n)  y (n  2)  y (n  1)  y (n)
6 6
V1  V2  83.15mV
Y ( z) 1
H ( z)  
X ( z ) 1  1 z 2  5 z 1
43. Two sequences [a, b, c] and [A, B, C] are related as 6 6
 A 1 1 1  a 1 1
2 Poles are at z  ,
 B   1 W 1 W 2   b  where W  e j 3 2 3
   3 3    3

C  1 W32 W34   c  45. The pole-zero diagram of a causal and stable
If another sequence [p, q, r] is derived as, discrete-time system is shown in the figure. The zero at
 p  1 1 1  1 0 0   A / 3 the origin has multiplicity 4. The impulse response of the
 q   1 W 1 W 2  0 W 2 0   B / 3 system is h[n]. If h [0] = 1, we can conclude
   3 3  3   
 r  1 W32 W34  0 0 W34  C / 3
then the relationship between the sequences [p, q, r] and
[a, b, c] is
A. [p, q, r] = [b, a, c] B. [p, q, r] = [b, c, a]
C.[p, q, r] = [c, a, b] D.[p, q, r] = [c, b, a]
Answer ||| C
 A  abc 
  
Solution ||| B  a  bW3  cW3
1 2 
A. h [n] is real for all n
   
C   a  bW32  cW31  B. h[n] is purely imaginary for all n

9|Page
www.gradeup.co

C. h [n] is real for only even n 1  s


D. h [n] is purely imaginary for only odd n Transfer function  ...(ii )
1   s
Answer ||| A
Where,
z4  = Lead time constant = R1C
Solution ||| H ( z ) 
 1  1  
2
1  1
2
R2
 z      z     And  
 2  4   2  4  R1  R2
1 1 1 1 Comparing equation (i) and (ii), we get
Poles =   j ,  j 1 1
2 2 2 2  and  
z 4
z4 2 4
  1
 2 1  2 1 1 2 Or  
 z  z   z  z   z  z  4  z
4 2
2
 2  2
RC time constant = 0.5
4
z
H ( z)  48. A plant transfer function is given as
1
z 
4

4  K  1
G(s)   K p  I  . When the plant operates in a
1  1  s  s ( s  2)
 Z 4   Z 4 1  z 4
4  4 unity feedback configuration, the condition for the
stability of the closed loop system is
1
Z4  KI
4 A. K P  0
2
1 B. 2 K I  K P  0

4
C. 2 K I  K P
46. The open-loop transfer function of a plant in a unity D. 2 K I  K P
K ( s  4) Answer ||| A
feedback configuration is given as G ( s )  .
( s  8)( s 2  9)  Ki  1 
Solution ||| G ( s )   K p    s ( s  2) 
The value of the gain K( > 0) for which –1 + j2 lies on  s  
the root locus is __________. The closed loop transfer function for unity feedback
A. 25.54 B. _26.34
C. 28.43 D. 84.23 G(s)
 2
 K p s  Ki 
Answer ||| A 1  G ( s) s ( s  2)  ( K p s  K i )
Solution |||
K p s  Ki
K ( s  2) 
If G(s) = , H(s) = 1 s  2s 2  K p s  Ki
3
( s  8)( s 2  9)
Using Routh's tabular form:
Make G  s  H  s   1 ⇒ K = 25.54
1
Or can be solved by use of characteristic equation S3 Kp
2
2
S K
47. A lead compensator network includes a parallel 1
2K p  Ki I
combination of R and C in the feed-forward path. If the S 0
0 2
transfer function of the compensator is S
Kp
s2
Gc ( s )  , the value of RC is _________. For system to be stable:
s4
Kp  0
A. 0.5 B. 0.7
C. 1.5 D. 0.8 2K p  K I
Answer ||| A And  0 Or 2 K p  K I  0
2
s2 K
Solution ||| GC ( s )  ....(i )
s4 Or K p  I  0
2
For lead compensator,
49. The input X to the Binary Symmetric Channel (BSC)
shown in the figure is '1' with probability 0.8. The cross-
over probability is 1/7. If the received bit Y= 0, the
conditional probability that '1' was transmitted is ______.

10 | P a g e
www.gradeup.co

A. 0.5 B. 0.4
C. 0.7 D. 0.3
Answer ||| B
Solution ||| P( x  1/ Y  0)
P(Y  0 / x  1) P( x  1)

P(Y  0 / X  1) P( X  1)  P(Y  0 / X  0) P( X  0)
1
  (0.8)
 7  0.4
 
1 6
  (0.8)    (0.2)
7 7

50. The transmitted signal in a GSM system is of 200 kHz


bandwidth and 8 users share a common bandwidth using
TDMA. If at a given time 12 users are talking in a cell, For z(t) = 10x(t), maximum value of W must be less than
the total bandwidth of the signal received by the base 350 Hz.
station of the cell will be at least (in kHz) _______ .
A. 300 kHz B. 400 kHz 52. A source emits bit 0 with probability 1/3 and bit 1
C.450 kHz D.500 kHz with probability 2/3. The emitted bits are communicate to
Answer ||| B the receiver decides for either 0 or 1 based on the
Solution ||| In the question it is given that GSM requires received value R. It is given that the conditional density
200 kHz for 8 users and uses TDMA scheme to function of R are as
accommodate them. Thus for the next users we will
required an extra of 200 kHz bandwidth. Thus 400 kHz
bandwidth is to be used.

51. In the system shown in Figure (a), m(t) is a low-pass


signal with bandwidth W Hz. The frequency response of
the band-pass filter H(f) is shown in Figure (b). If it is
desired that the output signal z(t) = 10x(t). The
maximum value of W (in Hz) should be strictly less than
_________.
The minimum decision error probability is
A. 0 B. 1/12
C. 1/9 D. 1/6
Answer ||| D
Solution |||
Given the conditional density function of R as

A. 450 B. 400
C. 350 D. 500
Answer ||| C
Solution ||| x(t )  m(t ) cos 2400 t
Decision error probability that receiver decides 0 for a
Thus y (t )  10 x (t )  x 2 (t )
transmitted bit 1 is
fR/1 (r = 0) = 1/6
Again, the decision error probability that receiver decides
1 for a transmitted bit 0 is
fR/0 (r = 1) = 1/4
Drawing the spectrum set spectrum of x(t) be Hence, the minimum decision error probability is fR/1 (r =
0) = 1/6

53. The longitudinal component of the magnetic field


inside an air-filled rectangular waveguide made of a
perfect electric conductor is given by the following
expression

11 | P a g e
www.gradeup.co

The cross-sectional dimensions of the waveguide are


given as a = 0.08 m and b = 0.033 m. The mode of
propagation inside the waveguide is 55. Consider a uniform plane wave with amplitude (E0) of
A. TM12 C. TE21 10 V/m and 1.1 GHz frequency travelling in air, and
B. TM21 D. TE12 incident normally on a dielectric medium with complex
Answer ||| B
Solution ||| relative permittivity  r  and permeability  r  as
shown in the figure.

Axial component is H ⇒ the propagating


Mode is TEmn, m, n can be found by

The magnitude of the transmitted electric field


component (in V/m) after it has travelled a distance of 10
cm inside the dielectric region is ________.
A. 0.2 B. 0.3
C.0.1 D.0.4
Answer ||| C
Solution ||| Given,

 The mode of propagation is TE21.

Attenuation constant of the medium is given by


54. The electric field intensity of a plane wave traveling in
free space is given by the following expression

In this field, consider a square area 10 cm x 10 cm on a Where


plane x + y = 1. The total time-averaged power (in mW)
passing through the square area is __________.
A. 43.29 B. 53.29
C.56.39 D.43.28
Answer ||| B
Solution |||

Power density

Time average power


At a distance of 10 cm |E| is given by
Pavg  P.ds
 1 (24)2   aˆ x  aˆ y 
 â x  .   10 2 
 2 120   2 
24
Pavg   53.29mW
2

***

12 | P a g e

You might also like